LSAT and Law School Admissions Forum

Get expert LSAT preparation and law school admissions advice from PowerScore Test Preparation.

User avatar
 Dave Killoran
PowerScore Staff
  • PowerScore Staff
  • Posts: 5853
  • Joined: Mar 25, 2011
|
#59770
Complete Question Explanation
(The complete setup for this game can be found here: lsat/viewtopic.php?t=26553)

The correct answer choice is (D)

This is an unusual List question since it addresses only two of the four benches, and those benches are not sequential. An application of the VFJ super-block and the WG not-block eliminates only answer choice (C) (V is with J instead of F). That is a frustrating result, because in normal List questions the application of the rules knocks off several, if not all, of the incorrect answers. Since the rule application has only eliminated one answer choice, the key must be in the two benches that are not listed, benches 1 and 3. And this general way of thinking is a powerful tool in many other games as well: if the variables that you are working with do not seem to solve the problem, consider the other variables yet to be placed, or as in this situation, the other spaces that are unlisted and have yet to be considered.

Answer choice (A) can be eliminated because the assignment of the given variables leaves no room for the VFJ super-block. If you are uncertain of this, take a moment to make a hypothetical with G and X assigned to bench 2 and H and Y assigned to bench 4. It immediately becomes apparent that F and J will have to be assigned to benches 1 and 3, a violation of the rules.

Answer choices (B) and (E) can both be eliminated for the same reason: the assignment of the respective variables ultimately forces W and G together on one of the benches, a violation of the rules. For answer choice (B), W and G would be assigned together to bench 1, and for answer choice (E), W and G would be assigned together to bench 3.

Since answer choices (A), (B), (C), and (E) have been eliminated, answer choice (D) must be correct.

Overall, this is a tough question to face at the end of the game. Unless you focus on mentally placing the VFJ block and WG not-block into benches 1 and 3, none of the remaining answers appears incorrect. For many students the only solution is to try each answer choice and work out a hypothetical that proves or disproves the answer. Of course this is very time consuming. If you are having time difficulties with the games section, it might be useful to skip this question once you realize how long it is going to take. You could simply guess among answer choices (A), (B), (D), and (E).

Get the most out of your LSAT Prep Plus subscription.

Analyze and track your performance with our Testing and Analytics Package.